¿Es el número de partículas un problema para formular la física estadística de una manera matemáticamente rigurosa?

Las cantidades como el potencial químico se pueden expresar como algo así

m = T ( S norte ) mi , V .

Ahora la entropía es el registro de algún volumen, que depende del número de partículas norte . Como en esta definición sumamos números naturales de partículas, ¿hay alguna buena forma de evaluar la derivada?

Lo que se hace prácticamente, es decir, cuando se trata de un gas ideal, es calcular la cantidad Γ norte , que podría resultar ser π norte / 2 ( norte / 2 ) ! , y luego se obtendrá una expresión S ( mi , norte , V ) que, por supuesto, puede tratarse como si fuera una función sobre R 3 . Incluso si eso supone que uno tiene una expresión cerrada que es una función norte . En principio, estaría bien con eso, si uno tiene una función determinada (o al menos el conjunto de valores para todos norte ) sobre una cuadrícula y un procedimiento para introducir más y más puntos de cuadrícula para obtener una mezcla más fina, entonces existe una noción de convergencia a una derivada. Pero aquí las N están claramente siempre separadas por al menos 1 valor; no importa cuántos N haya (límite termodinámico), la malla no se vuelve más fina entre dos puntos dados.

Puede definir la derivada como calcular la tasa de cambio promedio entre dos números de partículas norte y norte + d y decir S norte evaluado en norte da el mismo valor para todos norte en uno de los d -Intervalos, pero luego habría que postular cómo llegar a d en cada nueva situación. Esto podría superarse en situaciones muy específicas al obtener una fracción "razonable" del número de Avogadros, pero esto no es del todo matemático y los valores de los diferentes esquemas de aproximación de diferencias finitas siempre son diferentes.

En general, creo que no hay una comprensión categórica de lo que debe ser el espacio dimensional fraccional (espacio de fase en este caso) y, por lo tanto, el procedimiento de evaluación de la derivada debe postularse explícitamente.

Llegué a la conclusión de que usas la gran función de partición canónica Ξ ( β , m ) para cual norte = 1 β m en ( Ξ ( β , m ) ) para eliminar la variable continua m para el continuo norte . Tratando norte como parámetro y cambiar su valor equivale a cambiar m .

Respuestas (3)

No, no es un problema. La razón es que, para que expresiones como

m = T ( S norte ) mi , V .

para que tenga sentido, debe utilizar el gran conjunto canónico (o una generalización del mismo), en el que las partículas pueden entrar y salir del sistema. Como consecuencia, norte no representa un número entero de partículas, sino un valor esperado: norte = i pags i norte i , donde la suma es sobre todos los estados posibles del sistema, y norte i es el número de partículas cuando el sistema está en estado i . Dado que las probabilidades pags i son continuos, norte es continua también, y no hay ningún problema matemático al considerar un cambio arbitrariamente pequeño en ella.

La forma de hacer esto es notar que, para el conjunto grancanónico con una sola especie química,

pags i = 1 Z ( λ ) Exp ( λ tu tu i λ V V i λ norte norte i ) ,
dónde λ tu , a menudo denotado β , es 1 / T ; λ V = PAGS / T y λ norte = m / T , (he suprimido la constante de Boltzmann para mejorar la legibilidad), y λ indica un vector de los tres λ 's. Esto nos permite escribir la entropía como
S = i pags i Iniciar sesión pags i = i pags i ( Iniciar sesión Z ( λ ) + λ tu tu i + λ V V i + λ norte norte i ) = Iniciar sesión Z ( λ ) + λ tu tu + λ V V + λ norte norte ,
donde de nuevo el tu , V y norte sin subíndices indican los valores esperados. S ahora puede considerarse una función de estos valores esperados, lo que nos permite escribir
S ( tu , V , norte ) norte = λ norte = m / T ,
lo que nos da su expresión original.

Considerando S en función de las variables extensivas tu , V y norte en lugar de los intensivos λ Puede parecer un movimiento un poco extraño, pero tiene sentido si adopta el enfoque de MaxEnt para la mecánica estadística. En este formalismo la expresión anterior para pags i se obtiene maximizando i pags i Iniciar sesión pags i sujeto a las restricciones que los valores esperados tu , V y norte debe tomar valores particulares. En este caso es muy natural considerar S una función de sus valores, y Z una función sólo de la λ 's.

Me encuentro un poco escéptico con respecto a la afirmación "... para que expresiones como... tengan sentido, debe usar el gran conjunto canónico... en el que las partículas pueden entrar y salir del sistema". ¿Objeta la respuesta de Arnold Neumaier a continuación? Aunque ciertamente no hay problema en el gran conjunto canónico, ¿no es un poco fuerte decir que la derivada no tiene sentido en el límite termodinámico del conjunto microcanónico? ¿No obtengo la respuesta correcta en el límite térmico usando una energía fija y un número de partículas si uso S = k en Ω ?
@joshphysics tienes razón, esa declaración es demasiado fuerte, también es significativa en el límite termodinámico para el conjunto microcanónico. (En el límite, ambos conjuntos están de acuerdo a menos que esté en una transición de fase). Corregiré mi respuesta cuando vuelva a la computadora, si no recuerdo mal.

La fórmula que escribes es de termodinámica. En la versión de mecánica estadística, es válido en el gran conjunto canónico solo si interpreta las variables extensivas como valores esperados. (Consulte, por ejemplo, el capítulo 9 de mi libro en línea Classical and Quantum Mechanics via Lie algebras , arXiv:0810.1019 ). Pero los valores esperados son continuos incluso cuando el operador correspondiente tiene un espectro discreto.

Si en cambio trabaja con el conjunto canónico o microcanónico, la fórmula es válida solo en el límite termodinámico. En este límite, la variable termodinámica que tiene sentido en el límite no es norte pero norte ¯ = norte V ¯ / V , dónde V ¯ es un volumen de referencia finito que en las fórmulas termodinámicas toma el lugar de V . Otra vez, norte ¯ toma en el límite un continuo de valores.

Así, en cada una de las versiones típicamente cubiertas en mecánica estadística, todo es consistente, ya que no hay ningún problema teórico al tomar la derivada. Por supuesto, encontrar fórmulas cerradas para el límite termodinámico es la parte difícil, tratable solo en unos pocos casos, pero en estos casos uno puede verificar la diferenciabilidad excepto en los puntos de transición de fase.

Bien, entonces en el segundo párrafo dices que la variable que uno debe usar rigurosamente no es realmente norte , ¿Correcto? Con respecto al primer párrafo, para calcular la gran función de densidad del espacio de fase canónica, ya necesita el cambio del volumen del espacio de fase. Entonces, ¿cómo llega a eso, ya que necesita ese objeto para obtener los valores esperados?
Siempre que se trate de un límite termodinámico, norte va al infinito. El material real se considera como una pieza finita (del material infinitamente extendido con infinito norte ) de volumen %\overline V$. Esto explica mi fórmula. - El gran conjunto canónico ya funciona a un volumen finito; no es necesario tomar un límite (excepto para calcular realmente una función de partición). Véase, por ejemplo, mi libro para la derivación de las fórmulas termodinámicas del gran conjunto canónico. Un cambio de volumen significa integrar sobre una región de espacio de fase diferente.

El pensamiento de Gibbs sobre esto fue ( Principios elementales , nota al pie de la página 204) "Estrictamente hablando, ψ gramo mi norte no se determina en función de v 1 , v h , a excepción de los valores integrales de estas variables. Sin embargo, podemos suponer que está determinada como una función continua por cualquier proceso adecuado de interpolación. ψ gramo mi norte es la energía libre del conjunto canónico que incluye el conteo correcto de Boltzmann para partículas indistinguibles, y v i son números de diferentes especies de partículas.

Sin embargo, "cualquier proceso adecuado" es un poco vago. Podría construir un sistema hipotético en el que los números de partículas pares fueran ampliamente favorecidos sobre los números de partículas impares (por ejemplo, haciendo una penalización de energía para un número de partículas impar). Si intentara definir el potencial químico mediante una simple diferencia finita, me metería en problemas.

La respuesta (como señalaron Nathaniel y Arnold) es que cuando se permiten fluctuaciones en el número de partículas, es decir, en grandes conjuntos, el valor esperado del número de partículas varía continuamente y la derivada parcial es buena. Con el gran conjunto canónico , en particular, Gibbs demostró que tenemos un valor de potencial químico bien definido y útil que funciona hasta los sistemas microscópicos.

(En realidad, el problema del número de partículas discretas no es diferente del problema del valor de la energía discreta en los sistemas cuánticos. Una vez más, el conjunto canónico, ya sea grande o pequeño, proporciona un valor de temperatura perfectamente funcional incluso en sistemas microscópicos. Para tener una temperatura bien definida en un sistema microscópico, el precio es que debemos permitir que el conjunto involucre sistemas de una variedad de valores de energía).